LSAT and Law School Admissions Forum

Get expert LSAT preparation and law school admissions advice from PowerScore Test Preparation.

 Administrator
PowerScore Staff
  • PowerScore Staff
  • Posts: 8917
  • Joined: Feb 02, 2011
|
#23559
Complete Question Explanation

Assumption. The correct answer choice is (E)

Based upon our knowledge of the real world — as opposed to the crazy, mixed up LSAT world — we know that politicians often base their actions in office on the results of opinion polls. If Astorga's campaign promises are crafted primarily from opinion polls, who is to say that her actual actions in office will not follow the same trend. In this assumption question, we must find the answer choice that would eliminate that possibility, so as to allow us to get to the stimulus conclusion.

Answer Choice (A): Just because she will not be capable of carrying out her campaign promises, does not mean that she does not intend to make the attempt to carry them out. The stimulus is not addressing what she will actually accomplish, but what she intends to try to accomplish.

Answer Choice (B): The problem in the stimulus argument has nothing to do with the accuracy or inaccuracy of opinion polls. The stimulus is merely assuming that she will not continue to rely on those polls — regardless of their accuracy — once she is in office.

Answer Choice (C): The stimulus conclusion is not concerned with the chances of Astorga being elected, but with what will result if she is elected. Therefore, our assumption answer needs to deal with what will occur after the election, not whether her campaign promises will be successful.

Answer Choice (D): Whether or not Astorga has strong opinions of her own does not affect the issue of whether she will simply follow opinion polls if elected. Actually, if she has no strong opinions of her own, that would aim more towards a conclusion that she will simply follow the opinion polls if elected. Therefore she would be telling the voters what she actually intends to do.

Answer Choice (E): This is the correct answer choice. Within the stimulus, we have no link between what Astorga is promising and what she actually intends to do if elected. This answer choice provides that link. Now that we know that she does not intend to do what she said she would, we can get to the conclusion that the voters are being deceived.
 ellenb
  • Posts: 260
  • Joined: Oct 22, 2012
|
#10574
Dear Powerscore,

For this question, I have picked B, however, I know that it is E. I just want to know why E is the right one and why B is wrong.
I can break down my thought process if needed,
Conclusion is that: voters are not being told the what Astroga intends to do if she becomes mayor

Premise: Astroga has learned it from opinion polls what they want the new mayor to do.
Premise: Astroga just wants to please voters (I thought of this to be the conclusion for a minute)

Thanks

Ellen
 David Boyle
PowerScore Staff
  • PowerScore Staff
  • Posts: 836
  • Joined: Jun 07, 2013
|
#10607
ellenb wrote:Dear Powerscore,

For this question, I have picked B, however, I know that it is E. I just want to know why E is the right one and why B is wrong.
I can break down my thought process if needed,
Conclusion is that: voters are not being told the what Astroga intends to do if she becomes mayor

Premise: Astroga has learned it from opinion polls what they want the new mayor to do.
Premise: Astroga just wants to please voters (I thought of this to be the conclusion for a minute)

Thanks

Ellen
Hello,

B, "The opinion polls on which Astorga’s promises are based do not accurately reflect what voters want the new mayor to do", has nothing to do with Astorga's intentions.
E, "Astorga does not actually intend, if elected, to do what she has learned from the public opinion polls that voters want the new mayor to do", is correct because it links the polls with her intentions.

David
 ellenb
  • Posts: 260
  • Joined: Oct 22, 2012
|
#10642
So, for B we do not care what the polls reflect, it is that in E the assumptions says that she is sort of "faking it" saying she says she will do one thing where actually she is thinking of doing something else. It is sort of assuming that she is faking her true intentions and says that she will do what the voters want her to do when in actuality it is different.

thanks

Ellen
 David Boyle
PowerScore Staff
  • PowerScore Staff
  • Posts: 836
  • Joined: Jun 07, 2013
|
#10663
ellenb wrote:So, for B we do not care what the polls reflect, it is that in E the assumptions says that she is sort of "faking it" saying she says she will do one thing where actually she is thinking of doing something else. It is sort of assuming that she is faking her true intentions and says that she will do what the voters want her to do when in actuality it is different.

thanks

Ellen
Hello,

Basically, yes, E is saying she may be "faking", lying, etc.

David
 Morehouse20
  • Posts: 6
  • Joined: Feb 25, 2019
|
#63412
I put D for this question and I understand why the answer is E but I wanted to know more about identifying the conclusion which was in the first sentence if possible. I also have a question about whether negation should be utilized on all questionable assumption questions and if so what is the process?
 Adam Tyson
PowerScore Staff
  • PowerScore Staff
  • Posts: 5153
  • Joined: Apr 14, 2011
|
#63429
Let me see if I can help here, Morehouse20! First, here's how I go about identifying the conclusion: I simplify the claims and see how they fit together. So, here are the claims, in order, simplified:

1. she just's saying what they want to hear
2. she's saying what the polls tell her to say
3. she's not going to do that

I'm pretty sure that the second claim is not the conclusion, because the other claims - like "she's not going to do that" - don't really support it. So, it's either:

premise: she's saying what the polls tell her to say
premise: she just's saying what they want to hear
conclusion: therefore, she's not going to do that

or else it's:

premise: she's saying what the polls tell her to say
premise: she's not going to do that
conclusion: therefore, she just's saying what they want to hear

I think the latter structure makes more sense as an argument, especially because in the former setup we could just as easily have concluded that she IS going to do that. So, putting myself in the author's head here, I believe he is trying to prove that Astorga is just saying what people want to hear, which is the first sentence of the stimulus.

Now, as to your second question about using the Negation Technique: you should use it on any Assumption question (aka Necessary Assumption) where you find yourself with more than one contender, after having prephrased an answer choice and sorted the answers into losers and contenders. If you haven't prephrased, stop the presses and do that first, before venturing out into the answers. If you have only one contender, pick it! Don't waste time on negations, especially if the answer matches your prephrase.

I hope that helps!
 lsatstudying11
  • Posts: 54
  • Joined: Jul 30, 2020
|
#77627
Hello!

Would answer E also be correct if this were a Justify question? I sometimes have trouble distinguishing between the two, and this answer seems like it would justify or guarantee the conclusion.

Thank you in advance! :lol:
 Jeremy Press
PowerScore Staff
  • PowerScore Staff
  • Posts: 1000
  • Joined: Jun 12, 2017
|
#77696
Hi lsatstudying,

If you follow the structure of the argument outlined in Adam's post above, which I recommend you do (because I agree with him!), the conclusion is the first sentence: "Astorga’s campaign promises are apparently just an attempt to please voters."

If that conclusion is going to be true, then answer choice E has to be true: she can't be making those promises with the sincere intention of keeping them.

But that answer doesn't completely rule out other possible motivations behind Astorga's campaign promises. What if she has no intention of keeping the promises, but she's not JUST trying to please voters? What if she's making those promises not because she wants to please the voters, but because she's being forced to make them by her campaign advisors? Answer choice E doesn't rule that out, but that fact would undermine the conclusion. So answer choice E by itself doesn't justify the conclusion. To justify the conclusion, we'd need an answer that fully rules out ALL motivations EXCEPT that of pleasing voters.

I hope this helps!

Jeremy

Get the most out of your LSAT Prep Plus subscription.

Analyze and track your performance with our Testing and Analytics Package.